LSAT and Law School Admissions Forum

Get expert LSAT preparation and law school admissions advice from PowerScore Test Preparation.

User avatar
 katehos
PowerScore Staff
  • PowerScore Staff
  • Posts: 184
  • Joined: Mar 31, 2022
|
#96120
Hi emilyjmyer!

The interpretation of the rule is most similar to the last one you mentioned in which there cannot be a group without an M or a V. Hence why the best way to diagram this rule is M/V for the fall and spring (good job)!

To help further your understanding, let's unpack the rule: "In each semester, either Montreal or Vancouver or both must host a meeting."

In each semester indicates that this rule will apply to both the fall and the spring semesters. With this in mind, we see that either Montreal or Vancouver or both must host a meeting. So, in each semester either M or V or BOTH is required to host a meeting. The rule does not specify that it has to be the same city (M/V) in each semester, just that each semester has to have at least one of them.

Additionally, it still leaves open the possibility that either M or V is the repeated variable, though it does not say one of them has to be. You can also consider how, if the rule were to mean that whichever of these three options must be the one that is repeated across both semesters, there would be a significant issue. If we were to put both M and V in one semester under that interpretation, then we'd have to put both M and V in the other semester. This is not possible since it would violate the requirement that each city hosts at least one meeting (one city would have to be left out entirely to repeat both M and V)!

I hope this helps :)
Kate
User avatar
 SGD2021
  • Posts: 72
  • Joined: Nov 01, 2021
|
#97129
Hello,

Would someone possibly be able to explain why Kelsey Wood's template explanation uses Hf and H not in f (and V in f and V not in f) as the only two possibilities? How can we apply this idea in a different game?
 Adam Tyson
PowerScore Staff
  • PowerScore Staff
  • Posts: 5153
  • Joined: Apr 14, 2011
|
#97135
Because those options are very limiting, SGD, and they are, logically, exhaustive. Either H is in the Fall or it is not - there is no third alternative! Same with V in the Spring. Those two choices in combination are powerful because H in the Fall and V in the Spring are Sufficient Conditions in two of the rules that drive the game. Any time you have only two options for a variable, that because an interesting thing to explore for possibly doing templates, and when you have two such influential variables each with only two options, this approach could be fruitful.

Get the most out of your LSAT Prep Plus subscription.

Analyze and track your performance with our Testing and Analytics Package.